The braking distance, D, of a car is directly proportional to the square of its speed, v. When d=5, v=10
Find d when v=70​

Answers

Answer 1

Answer:

         d = 245

Step-by-step explanation:

d is directly proportional to the square of a speed v

d = av²

5 = a•10²

5 = 100a

a = 0.05

d = 0.05v²

d = 0.05•70²

d = 0.05•4900

d = 245


Related Questions

A Water flows through a pipe at a rate of 10 milliliters every 8.5 seconds. Express this
rate of flow in liters per minute. Round your answer to the nearest hundredth

Answers

Answer:

The answer to the nearest hundredth is 0.07 liters per minute

Step-by-step explanation:

In this question, we are told to express the given metric in liters per minute.

The key to answering this question, is to

have the given measurements in the metric in which we want to have the answer.

Hence, we do this by converting milliliters to liters and seconds to minute.

Let’s start with milliliters;

Mathematically;

1000 milliliters = 1 liters

10 milliliters = x liters

x * 1000 = 10 * 1

x = 10/1000

x = 1/100

x = 0.01 liters

For the seconds;

We need to convert the seconds to minutes;

Mathematically;

60 seconds = 1 minute

8.5 seconds = y minutes

60 * y = 8.5 * 1

y = 8.5/60

y = 0.14167 minutes

Now, our rate of flow is liters per minute, that means we have to divide the volume by the time;

Hence, we have ;

0.01/0.14167 = 0.070588235294

Which to the nearest hundredth is 0.07

how to find the theta with side lengths of a triangle

Answers

Step-by-step explanation:

Hello, there!!!

I hope you mean the question is like the above problem in picture.

so, let's simply work with it.

here, we may use cosine rule,

so, according to cosine rule,

[tex] {c}^{2} = {a}^{2} + {b}^{2} - 2ab.cosc[/tex]

so, just put value of formulae here,

we get;

5^2 = 3^2 + 4^2 - (2×3×4) . cos thita

or, 25 = 9 + 16 -24 cos thita.

or, 24 cos thita = 0

or, cos thita = 0/25

or, cos thita = 0

now, taking cos to right side we get,

[tex] {cos}^{ - 1 } (0)[/tex]

now, after typing cos ^-1 (0) we get angle as 90°.

(note: in step {cos thita = 0} you couold directly write like; cos thita = cos 90°. and cos would be cancelled in it as cos 90°=0. but it is only applied in particular angle like 0°,30°,60°,..... which are identified or if you don't know you must use the method above using calculator and remember to put inverse {cos^-1}).

so, In this way we find angle.

I hope it helps....

What is the y−intercept of the line that passes through the point (4,9)and is parallel to the line y=12x+2?

Answers

Answer:

y- intercept = - 39

Step-by-step explanation:

The equation of a line in slope- intercept form is

y = mx + c ( m is the slope and c the y- intercept )

y = 12x + 2 ← is in slope- intercept form

with slope m = 12

Parallel lines have equal slopes, thus

y = mx + c ← is the partial equation

To find c substitute (4, 9) into the partial equation

9 = 48 + c ⇒ c = 9 - 48 = - 39 ← y- intercept

Answer:

y-intercept = -39

Step-by-step explanation:

if two lines are parallel it means they have the same gradient so we compare the equation given to the default equation of a line

y=mx+c

y=12x+2

comparing we have the gradient m=12 now finding the equation of the line parallel to the given line we use

y-y1=m(x-x1)

y1=9 and x1=4

y-9=12(x-4)

y-9=12x-48

y=2x-48+9

y=2x-39

comparing to the default equation of a line y=mx+c where c is the y-intercept

therefore the y-intercept is -39

Wait times at a dentist's office are typically 21 minutes, with a standard deviation of 2 minutes. What percentage of people should be seen by the doctor between 17 and 25 minutes for this to be considered a normal distribution?

Answers

ANSWER: 95%

HOW:
95% of a group of data with a normal distribution is between two standard deviations to the right and left

Answer:

95%

Step by step explanation:

z = 17-21 / 2 and z = 25-21/2

z=-2 (2.28%) z=2 (97.72%)

97.72 - 2.28 = 5.44

100% - 5.44% is about equal to 95%

What is m Round the answer to the nearest whole number.
O 30°
O 35°
O 55°
O 60°

Answers

Answer:

30

Step-by-step explanation:

fufyfuf7fjcjcufuy7fufucyyxyvkbuvufudydy shut up

Please help me understand this. Thank you! Gina has borrowed 100 songs from her friend. She plans to download an equal number of songs on her music player each week for 5 weeks. The graph shows the number of songs left to download, y, for a certain number of weeks, x: A graph titled Song Downloading shows the Number of Weeks on x-axis and Number of Songs Left to Download on the y-axis. The x-axis scale is shown from 0 to 5 at increments of 1, and the y-axis scale is shown from 0 to 140 at increments of 20. A straight line joins the ordered pairs 0, 100 and 1, 80 and 2, 60 and 3, 40 and 4, 20 and 5, 0. Part A: What is the rate of change and initial value of the function represented by the graph, and what do they represent in this scenario? Show your work to find the rate of change and initial value. (6 points) Part B: Write an equation in slope-intercept form to model the relationship between x and y. (4 points)

Answers

Answer:

Rate of Change/Slope = -20

Equation: y= -20x +100

Step-by-step explanation:

A. We know the rate of change is also known as the slope.  If we used the slope formula to find the slope we can find the Rate of Change.

[tex]\frac{y2 - y1}{x2 - x1} = \frac{100-80}{1-2} = \frac{20}{-1} = -20[/tex]

B. Since we know the slope and 1 point on the graph we can substitute them in for 'b'

(0,100)

(100)=-20(0) + b

b = 100

Since we know the slope and the b value we can write the equation:\

y = -20x +100

The rate of change refers to the slope of the line, that is the change in y-axis per unit change in the value on the x-axis. Hence, the rate of change is :

Slope = - 20y = -20x + 100

Slope = Rise / Run

Rise = (y2 - y1) = (0 - 100) = - 100

Run = (x2 - x1) = (5 - 0) = 5

Slope = - 100 / 5 = - 20

General form of a slope - intercept relation :

y = bx + c

The intercept, c can be calculated thus:

100 = - 20(0) + c

100 = 0 + c

c = 100

Hence, the slope - intercept equation will be y = - 20x + 100

Learn more : https://brainly.com/question/18479471

Which of the following are natural numbers? There may be more than one correct answer. Select all that apply. If only one answer is correct, select "only" and the answer that applies. A.) only B.) −1,−2,−3,… C.) 7,8,9,… D.) fractions E.) 22

Answers

Answer:

Option C and option E

Step-by-step explanation:

C.) 7,8,9,…

E.) 22

Natural numbers are also called counting numbers.

They begin from 1, 2, 3 to infinity.

Natural numbers are greater than zero (0)

They do not have decimal point in them.

They are positive integers, as such they do not have minus

Natural numbers can include commas when they are large like 3,000

construct a right-angled triangle ABC where angle A =90 degree , BC= 4.5cm and AC= 7cm. please ans fast........ Very urgent. Pls don't give wrong answers

Answers

Answer and Step-by-step explanation: The described right triangle is in the attachment.

As it is shown, AC is the hypotenuse and BC and AB are the sides, so use Pytagorean Theorem to find the unknown measure:

AC² = AB² + BC²

[tex]AB^{2} = AC^{2}-BC^{2}[/tex]

[tex]AB =\sqrt{AC^{2}-BC^{2}}[/tex]

[tex]AB =\sqrt{7^{2}-4.5^{2}}[/tex]

[tex]AB =\sqrt{28.75}[/tex]

AB = 5.4

Then, right triangle ABC measures:

AB = 5.4cm

BC = 4.5cm

AC = 7cm

Please answer this question now

Answers

Answer:

30.9 cm²

Step-by-step explanation:

To find the surface area of this figure, we find the area of the base and the 3 identical sides.

The base is split into two identical right triangles. Let's find the area of one and multiply by two.

Half of 3: 1.5

[tex]1.5\cdot2.6=3.9\\3.9\div2=1.95[/tex]

There are two right triangles:

[tex]1.95\cdot2=3.9[/tex]

The area of one of the sides will be the same thing, except the height is 6.

[tex]1.5\cdot6=9\\9\div2=4.5\\4.5\cdot2=9[/tex]

There are 3 sides identical to this one:

[tex]9\cdot3=27[/tex].

Add 27 and 3.9:

[tex]27+3.9=30.9[/tex]

Hope this helped!

Answer:

30.9 square centimeters

Step-by-step explanation:

3 * 1/2(3)(6) + 1/2(3)(2.6) = 30.9

need help please. Will give you 5-stars and a big thank you comrades

Answers

Answer:

first answer

Step-by-step explanation:

(8x³ - 22x² - 4) / (4x - 3)

when you do long division you get the first answer

HELP ASAP ITS SO HARD! Kelsey did the following division problem. Her teacher says that the quotient she found is wrong. −2 5/6 ÷ 1 1/3 −17/6 ÷ 4/3 −6/17• 3/4 −6×3 divided by 17×4 −18/68 −9/34 A. Identify what Kelsey did wrong in her calculations. B. Find the correct quotient, showing all of your calculations.

Answers

Part A

Her steps were

[tex]-2 \frac{5}{6} \div 1 \frac{1}{3}\\\\-\frac{17}{6} \div \frac{4}{3}\\\\-\frac{6}{17} \times \frac{3}{4}\\\\-\frac{6\times 3}{17\times4}\\\\-\frac{18}{68}\\\\-\frac{9}{34}\\\\[/tex]

Kelsey made a mistake on line 3. Note how the 17/6 flips to 6/17. This is not correct. You keep the first fraction the same, but you do flip the second fraction. This only applies when you divide two fractions.

The third step should look like [tex]-\frac{17}{6}\times \frac{3}{4}[/tex]

=======================================================

Part B

Here's what she should have written

[tex]-2 \frac{5}{6} \div 1 \frac{1}{3}\\\\-\frac{17}{6} \div \frac{4}{3}\\\\-\frac{17}{6} \times \frac{3}{4}\\\\-\frac{17\times 3}{6\times 4}\\\\-\frac{51}{24}\\\\-\frac{17}{8}\\\\[/tex]

If you want to convert that improper fraction to a mixed number, then you could do something like this

[tex]-\frac{17}{8} = -\frac{16+1}{8}\\\\-\frac{17}{8} = -\frac{16}{8}-\frac{1}{8}\\\\-\frac{17}{8} = -2 \frac{1}{8}\\\\[/tex]

Or you could divide 17 over 8 using long division to get 2 remainder 1. The 2 is the quotient that goes to the left of the 1/8. The remainder of 1 is the numerator of 1/8.

Show all work to solve the equation for x. If a solution is extraneous, be sure to identify it in your final answer.

square root of the quantity x minus 3 end quantity plus 5 equals x

Answers

Answer:

Step-by-step explanation:

[tex]\sqrt{x-3} +5=x\\\sqrt{x-3} =x-5\\squaring ~both~sides\\x-3=x^2-10x+25\\x^2-10x-x+25+3=0\\x^2-11x+28=0\\x^2-7x-4x+28=0\\x(x-7)-4(x-7)=0\\(x-7)(x-4)=0\\x=7,4[/tex]

put x=7 in the given equation

[tex]\sqrt{7-3} +5=7\\\sqrt{4} +5=7\\2+5=7\\7=7[/tex]

which is true .

∴ x=7 is a solution of the given eq.

now put x=4 in the given eq.

[tex]\sqrt{4-3} +5=7\\1+5=7\\6=7\\[/tex]

which is not true.

∴x=4 is an extraneous solution.

a hotel manager wants miriam to tile their lobby using the dame design she created for Mr.Rivera.The lobby measures 45 feet by 45 feet. he wants the outer edge to be the same color as the center tile. will this occur ? justify your answer

Answers

Answer:

Yes it will occur

Step-by-step explanation:

The lobby measures 45 feet by 45 feet

Area of the lobby = 45 * 45

=2025 ft^2

So, the lobby has 2025 tiles

subtract 1 black tile in the center

2025 tiles - 1 black tile =2024 tiles

The number of blue tiles and black tiles is 2024 tiles

He wants the outer edge to be the same color as the center tile so, divide by 2

2024/2 = 1012 tiles

The number of tiles in the outer edge is 1012 tiles and the number of tiles in the center is 1012 tiles

Complete the table. At least the first few so I understand how to do it

Answers

Answer:

What we need to do is simply multiply the values in both columns e.g 4 * 3/36 = 12/36

Please check explanation for complete answer

Step-by-step explanation:

Here, we are concerned about filling the empty columns of the table.

What we want to do here is simply straightforward. All we need to do is to

multiply the values of x by the values of P(x) in each of the individual rows.

Also recall, we do not need to reduce the fractions.

So we have;

2. 3 * 2/36 = 6/36

3. 4 * 3/36 = 12/36

4. 5 * 4/36 = 20/36

5. 6 * 5/36 = 30/36

6. 7 * 6/36 = 42/36

7. 8 * 5/36 = 40/36

8. 9 * 4/36 = 36/36

9. 10 * 3/36 = 30/36

10. 11 * 2/36 = 22/36

11. 12 * 1/36 = 12/36

I need a lot of help

Answers

To add fractions with different denominators you must find the highest common factor (the highest number they both go into).

For 1 - The highest common factor is 8, 2x4 = 8, 4x2 = 8

now, whatever you do to the bottom, you must do to the top.

So:

3 x 2 = 6 and 5 x 4 = 20

Therefore, your answer would be 6/8 + 20/8

You do that for the rest of them as well, do you get it?

Answer:

3/4 + 5/2 = 3/4 + 10/4 = (3+10)/4 = 13/4  

    3.  4/15 + 4/5 = 4/15 + 12/15 = (4+12)/15 = 16/15

    5.  2/3  + 7/10 = 20/30 + 21/30  = (20+21)/30 = 41/30

Find f(x) and g(x) so the function can be expressed as y = f(g(x)). (1 point) [tex]y=\frac{7}{x^{2} } +10[/tex]

Answers

Answer:

The functions are [tex]f(x) = 7\cdot x+10[/tex] and [tex]g(x) = \frac{1}{x^{2}}[/tex], respectively.

Step-by-step explanation:

Let suppose that [tex]g(x) = \frac{1}{x^{2}}[/tex], then [tex]f(g(x))[/tex] is:

[tex]f(g(x)) = 7\cdot \left(\frac{1}{x^{2}} \right) + 10[/tex]

[tex]f(g(x)) = 7\cdot g(x) + 10[/tex]

Thus,

[tex]f(x) = 7\cdot x + 10[/tex]

The functions are [tex]f(x) = 7\cdot x+10[/tex] and [tex]g(x) = \frac{1}{x^{2}}[/tex], respectively.

Name a real world context to describe the sums of rational numbers.

Answers

Step-by-step explanation:

when you are cooking you need to measure fractions of ingredients

ASAP PLZ ANSWER!!! Can you tell me step by step to this question 8,595 ÷ 24?

Answers

Answer:

358 and remainder of 3

Step-by-step explanation:

1. Divide it like any other problem

24 goes into 85, 3 times with 13 left overBring down the 9 and 24 goes into 139, 5 times with 19 left overThen bring down the 5 and 24 goes inside 195, 8 times with 3 left overSo your remainder would be 3

Hope this helps

stagg high school has a rectangular swiming pool the area of the water in the pool is 1,800 meters squared the length is twice the width what is the perimeter of the pool find the length and width. SHOW WORK

Answers

Step-by-step explanation:

L*b=1800m^2

L=2b

2b*b=1800

2b^2=1800

b^2=900

b=30m

L=2*30

=60m

Perimeter=2(l+b)

=2(60+30)

=2*90

=180m

what is the answer for 6x-4=-26+5x

Answers

Answer: x=-22

Step-by-step explanation:

    6x-4=-26+5x

6x-4-5x=-26+5x-5x ⇔ subtraction property of equality

       x-4=-26

  x-4+4=-26+4 ⇔ addition property of equality

         x=-22

Answer:

x = - 22

Step-by-step explanation:

6x - 4 = - 26 + 5x

First of all group like terms

Send the constants to the right side of the equation and those with variables to the left

That's

6x - 5x = 4 - 26

Simplify

We have the final answer as

x = - 22

Hope this helps you

Which of the following symbols could correctly finish the statement. Select all that apply. 0___-8 = ≠ > < ≥ ≤

Answers

Answer:

>

Step-by-step explanation:

Even though its 0 its still greater than any negative number.

Answer:

Step-by-step explanation:

help me im dangered plzzzzzzzzzzzzzzzzzzzz

Answers

Answer:

A

Step-by-step explanation:

Hi!

An exponent is the same thing as just multiplying the expression by itself the number of times the exponent says. So we need to multiply 1/3 by itself three times.

1/3 * 1/3 * 1/3 = 1/27

the answer to your question is A

help help help me plZZZZZ ill give you brainly ;DDD

Answers

Answer:

the answer is 60.7

Step-by-step explanation:

60 to has a between numbers like given in the picture

so as number line it's

60.1 . 60.2 60.3 60.4 60.5 60.6 60.7 60.8 and continue

if u get any 3 digit number like 600 to 650 in number line

u do it like it the same 600.1 600.2.... and go on

Answer:

63½ or 63.5

Step-by-step explanation:

65-60=5

10points=5

1point=?

1×5/10= ½

that means the sequence continues after adding ½ i.e

60..60½...61...61½...62...62½...63...63½...64..64½...65

you have been asked the 8th number which is 63½

Joaquin used two types of flour in a muffin recipe. How much flour did he use in all? Solve any way you choose.

Answers

Answer:

3 1/6 cups of flour.

Step-by-step explanation

what equation accurately represent this statement three less than 4 times a number is less than 12

Answers

Answer: 4t - 3 < 12

Step-by-step explanation:

PLEASE HELP Ruri is a 30-year-old math teacher. She has been informed that she is the winner of a grand prize for the lottery. She can choose either a one-time payment of $20 million or $5000 per week for the rest of her life. Which choice would most likely result in the greatest amount of winnings for Ruri? Explain your reasoning.

Answers

Answer:

$5,000 per week

Step-by-step explanation:

Ruri is a 30 year old female.

there are about 4 weeks per month

there are about 52 weeks per year

52*5000 = 260,000

She would get 260,000 per year and lets see how much she would have at 40.

260,000*10

at 40 she would have 2,600,000

2,600,000*10

at 50 she would have 26,000,000

at 50 she already has earned more money that the $20 million.

She should go with the $5000 per week if she would like more money.

**Yoxelt buys 4 1/2 gallons of soda. One-fourth of the soda he bought was Pepsi and the rest was Sprite. How many gallons of Pepsi did Yoxelt buy? Show all work below.

Answers

Answer:

  1 1/8

Step-by-step explanation:

1/4 of the 4 1/2 gallons were Pepsi, so the amount is ...

  (1/4)(9/2) = (1·9)/(4·2) = 9/8 = 1 1/8

Yoxelt bought 1 1/8 gallons of Pepsi.

-7p+2(5p-8)=6(p+6)-7

Answers

Answer:

-15

Step-by-step explanation:

-7p+10p-16=6p+36-7

3p-16=6p+29

3p-6p=29+16

-3p=45

p=45/-3

p=-15

P=5

-7p+2(5p-8)=6(p+6)-7
Distribute
-7p+10p-16=6p+36-7
Add -7p and 10p together
3p-16=6p+36-7
Subtract 7 from 36
3p-16=6p+29
+6p (on both sides)
9p-16=29
+16 (on both sides)
9p=45
Divide both sides by 9
P=5

a second degree equation in one variable example how many solutions does it have ?a second degree equation in one variable example how many solutions does it have ? is it possible to have many solutions or no solutions tions give an example for each

Answers

Answer:

  0, 1, or 2 real solutions

Step-by-step explanation:

Including complex and repeated solutions, a polynomial with real coefficients, and of degree n, always has n solutions.

If you're only concerned about real solutions, a 2nd degree equation in one variable may have 0, 1, or 2 real solutions. Here are some examples.

0 solutions: x^2 +1 = 01 solution: x^2 = 02 solutions: x^2 -1 = 0

Which data set matches the box-and-whisker plot?
A) 12 13 15 19 23 23 25 26.5 28 30
B) 15 13 19 21 23 24 27 29 32
C) 11 31 13 15 19 21 21 25 27 29 31
D) 11 13 15 19 23 23 24 26.5 28 33​

Answers

Answer:

D) 11 13 15 19 23 23 24 26.5 28 33​

Step-by-step explanation:

The box-and-whisker plot displayed above has the following key values that we can use to identify which of the given data set it matches. It has:

Minimum value = 11

Q1 = 15

Median = 23

Q3 = 26

Maximum value = 33

From the options given, using just the max and min value, we can conclude that the data set in option D matches the box plot.

The data set in option D has a minimum value of 11, and a maximum value of 33.

Other Questions
Find the area of the shape shown below.9.03.5252.units 7.. Getaway Travel Company reported net income for 2021 in the amount of $50,000. During 2021, Getaway declared and paid $2,000 in cash dividends on its nonconvertible preferred stock. Getaway also paid $10,000 cash dividends on its common stock. Getaway had 40,000 common shares outstanding from January 1 until 10,000 new shares were sold for cash on July 1, 2021. A 2-for-1 stock split was granted on July 5, 2021. What is the 2021 basic earnings per share Please answer asap this person made a mistake what is the error and correct solution to this problem There are resume templates available on the internet to help you get started ? True or false multiply 2x^2-4x 6c^2-5x Why was a large Hindu temple constructed in the city of Cary, NC? Check all that apply. to offer educational opportunities for women and girls to provide a place for festivals and religious ceremonies to promote Indian music, art, dance, and culture to provide a safe place for Montagnard refugees to accommodate the growing Indian population Little Tots Gym has a required rate of return of 13%. The gym is considering the purchase of $12,500 of new equipment. The internal rate of return on the project has been calculated to be 11%. This project:________ On December 31, 2016, when its Allowance for Doubtful Accounts had a debit balance of $1,432, Sunland Company estimates that 9% of its accounts receivable balance of $105,900 will become uncollectible and records the necessary adjustment to Allowance for Doubtful Accounts. On May 11, 2017, Sunland Company determined that B. Jareds account was uncollectible and wrote off $1,091. On June 12, 2017, Jared paid the amount previously written off.Required:Prepare the journal entries on December 31, 2016, May 11, 2017, and June 12, 2017. Combine like terms to simplify the expression: -5.55-8.55c+4.35c How do I solve 3(x+1)=5(x2)+7? Luke owns a trucking company. For every truck that goes out, Luke must pay the driver $17 per hour of driving and also has an expense of $1.75 per mile driven for gas and maintenance. On one particular day, the driver drove an average of 40 miles per hour and Luke's total expenses for the driver, gas and truck maintenance were $522. Write a system of equations that could be used to determine the number of hours the driver worked and the number of miles the truck drove. Define the variables that you use to write the system. Which kind of research is most likely to improve the health of theenvironment? WILL GIVE BRAINLIEST!!! Please answer this correctly without making mistakes Read the following sentence.Industrialized nations are taking strict measures to reduce greenhouse gas emissionsWhich word could be substituted for taking, if the speaker wanted to express a negative viewpointimposingrecommendingconsideringadvising Which structures are highlighted? Which structures are highlighted? pulmonary arterioles alveoli bronchioles pulmonary capillaries To what was Gustave Courbet's desire to paint individuals and scenes from ordinary life in the grand artistic tradition usually reserved for gods and royalty linked Describe the Pueblos (quick summary would be extremely helpful!) what is the term for a space between two teeth? The coffee cups can hold 7/9 of a pint of liquid. If Emily pours 2/3 of a pint of coffee into a cup,how much milk can a customer add? PLZ HELP!